Đến nội dung

Hình ảnh

$u_n=\frac{n+1}{2^{n+1}}.\sum \frac{2^k}{k}$

- - - - -

  • Please log in to reply
Chủ đề này có 3 trả lời

#1
Sagittarius912

Sagittarius912

    Trung úy

  • Thành viên
  • 776 Bài viết

Cho dãy số $\left \{ u_n \right \}$ với $u_n=\frac{n+1}{2^{n+1}}.(\frac {2}{1}+\frac{2^2}{2}+\frac{2^3}{3}+...+\frac{2^n}{n})$

Chứng minh rằng $\left \{ u_n \right \}$ có giới hạn và tìm $\lim_{n\rightarrow +\infty}u_n$

__________

Bài này dùng Stolz  được không m.n?


Bài viết đã được chỉnh sửa nội dung bởi Sagittarius912: 26-03-2013 - 16:12


#2
Crystal

Crystal

    ANGRY BIRDS

  • Hiệp sỹ
  • 5534 Bài viết

Cho dãy số $\left \{ u_n \right \}$ với $u_n=\frac{n+1}{2^{n+1}}.(\frac {2}{2}+\frac{2^2}{2}+\frac{2^3}{3}+...+\frac{2^n}{n})$

Chứng minh rằng $\left \{ u_n \right \}$ có giới hạn và tìm $\lim_{x\rightarrow +\infty}u_n$

__________

Bài này dùng Stolz  được không m.n?

 

Đề bài phải là $\left \{ u_n \right \}$ với $u_n=\frac{n+1}{2^{n+1}}.(\frac {2}{1}+\frac{2^2}{2}+\frac{2^3}{3}+...+\frac{2^n}{n})$ và $\lim_{n\rightarrow +\infty}u_n$

 

Lời giải:

Ta có: \[{u_{n + 1}} = \frac{{n + 2}}{{{2^{n + 2}}}}\sum\limits_{k = 1}^{n + 1} {\frac{{{2^k}}}{k}}  = \frac{{n + 2}}{{{2^{n + 2}}}}\left( {\frac{{{2^1}}}{1} + \frac{{{2^2}}}{2} + ... + \frac{{{2^{n + 1}}}}{{n + 1}}} \right)\]
$$ = \frac{{n + 2}}{{2\left( {n + 1} \right)}}.\frac{{n + 1}}{{{2^{n + 1}}}}\left( {\frac{{{2^1}}}{1} + \frac{{{2^2}}}{2} + ... + \frac{{{2^n}}}{n}} \right) + \frac{{n + 2}}{{2\left( {n + 1} \right)}} = \frac{{n + 2}}{{2\left( {n + 1} \right)}}\left( {{u_n} + 1} \right)$$
Tương tự: $${u_{n + 2}} = \frac{{n + 3}}{{2\left( {n + 2} \right)}}\left( {{u_{n + 1}} + 1} \right)$$
Từ đó: $${u_{n + 2}} - {u_{n + 1}} = \frac{{\left( {n + 1} \right)\left( {n + 3} \right)\left( {{u_{n + 1}} + 1} \right) - {{\left( {n + 2} \right)}^2}\left( {{u_n} + 1} \right)}}{{2\left( {n + 1} \right)\left( {n + 2} \right)}}$$
$$ = \frac{{\left( {{n^2} + 4n + 3} \right)\left( {{u_{n + 1}} - {u_n}} \right) - {u_n} - 1}}{{2\left( {n + 1} \right)\left( {n + 2} \right)}}$$
Uuy ra $\left\{ {{u_n}} \right\}$ là một dãy lượng giác. Do đó tồn tại $\mathop {\lim }\limits_{n \to  + \infty } {u_n} = L$.
 
Từ ${u_{n + 1}} = \frac{{n + 2}}{{2\left( {n + 1} \right)}}\left( {{u_n} + 1} \right)$ chuyển qua giới hạn, ta được: $L = \frac{1}{2}\left( {L + 1} \right) \Leftrightarrow L = 1$
 
Vậy $\mathop {\lim }\limits_{n \to  + \infty } {u_n} = 1$.


#3
Crystal

Crystal

    ANGRY BIRDS

  • Hiệp sỹ
  • 5534 Bài viết
Bài này bạn cũng có thể sử dụng Định lí Stolz với $\left\{ \begin{array}{l} {x_n} = \frac{2}{1} + \frac{{{2^2}}}{2} + ... + \frac{{{2^n}}}{n}\\ {y_n} = \frac{{{2^{n + 1}}}}{{n + 1}} \end{array} \right.$


#4
phudinhgioihan

phudinhgioihan

    PĐGH$\Leftrightarrow$TDST

  • Biên tập viên
  • 348 Bài viết

Cho dãy số $\left \{ u_n \right \}$ với $u_n=\frac{n+1}{2^{n+1}}.(\frac {2}{1}+\frac{2^2}{2}+\frac{2^3}{3}+...+\frac{2^n}{n})$

Chứng minh rằng $\left \{ u_n \right \}$ có giới hạn và tìm $\lim_{x\rightarrow +\infty}u_n$

__________

Bài này dùng Stolz  được không m.n?

 

 

Và hơn thế nữa, như ở đây : http://diendantoanho...s-k0nbinomnk-1/

 

Ta đã chứng minh được rằng $\sum_{k=0}^n \binom{n}{k}^{-1}=\dfrac{n+1}{2^{n+1}} \left(\dfrac{2}{1}+\dfrac{2^2}{2}+...+\dfrac{2^n}{n}+\dfrac{2^{n+1}}{n+1} \right)=u_n+1 $

 

Vậy $u_n=\sum_{k=1}^n \binom{n}{k}^{-1 }$


Bài viết đã được chỉnh sửa nội dung bởi phudinhgioihan: 26-03-2013 - 14:35

Phủ định của giới hạn Hình đã gửi

Đó duy sáng tạo ! Hình đã gửi


https://phudinhgioihan.wordpress.com/




0 người đang xem chủ đề

0 thành viên, 0 khách, 0 thành viên ẩn danh